A viewer suggested integral.

Поделиться
HTML-код
  • Опубликовано: 5 сен 2024
  • We evaluate a nice integral suggested by a viewer.
    Playlist: • Interesting Integrals
    Please Subscribe: www.youtube.co...
    Merch: teespring.com/...
    Personal Website: www.michael-pen...
    Randolph College Math: www.randolphcol...
    Randolph College Math and Science on Facebook: / randolph.science
    Research Gate profile: www.researchga...
    Google Scholar profile: scholar.google...
    If you are going to use an ad-blocker, considering using brave and tipping me BAT!
    brave.com/sdp793
    Buy textbooks here and help me out: amzn.to/31Bj9ye
    Buy an amazon gift card and help me out: amzn.to/2PComAf
    Books I like:
    Sacred Mathematics: Japanese Temple Geometry: amzn.to/2ZIadH9
    Electricity and Magnetism for Mathematicians: amzn.to/2H8ePzL
    Abstract Algebra:
    Judson(online): abstract.ups.edu/
    Judson(print): amzn.to/2Xg92wD
    Dummit and Foote: amzn.to/2zYOrok
    Gallian: amzn.to/2zg4YEo
    Artin: amzn.to/2LQ8l7C
    Differential Forms:
    Bachman: amzn.to/2z9wljH
    Number Theory:
    Crisman(online): math.gordon.edu...
    Strayer: amzn.to/3bXwLah
    Andrews: amzn.to/2zWlOZ0
    Analysis:
    Abbot: amzn.to/3cwYtuF
    How to think about Analysis: amzn.to/2AIhwVm
    Calculus:
    OpenStax(online): openstax.org/s...
    OpenStax Vol 1: amzn.to/2zlreN8
    OpenStax Vol 2: amzn.to/2TtwoxH
    OpenStax Vol 3: amzn.to/3bPJ3Bn
    My Filming Equipment:
    Camera: amzn.to/3kx2JzE
    Lense: amzn.to/2PFxPXA
    Audio Recorder: amzn.to/2XLzkaZ
    Microphones: amzn.to/3fJED0T
    Lights: amzn.to/2XHxRT0
    White Chalk: amzn.to/3ipu3Oh
    Color Chalk: amzn.to/2XL6eIJ

Комментарии • 141

  • @goodplacetostop2973
    @goodplacetostop2973 3 года назад +190

    14:09

    • @i.am.jihoonk
      @i.am.jihoonk 3 года назад +2

      who are you???

    • @ld1ego_733
      @ld1ego_733 3 года назад +13

      Where can I suggest integrals to Michael?😀

    • @goodplacetostop2973
      @goodplacetostop2973 3 года назад +13

      @@i.am.jihoonk I’m just a memer

    • @goodplacetostop2973
      @goodplacetostop2973 3 года назад +7

      @@ld1ego_733 Post in the comments, but Michael can miss it so you may have to repost. I know some people who kept posting the same suggestion for several days and eventually got the video.

    • @MichaelPennMath
      @MichaelPennMath  3 года назад +101

      @Good Place to Stop is right. You can also email me -- i won't list my email here, but it is easy enough to find online. The one bad thing with this channel getting bigger is that I tend to miss lots of nice suggestions, especially when I am busy with other things. I promise it is never due to a lack of my interest or gratitude though. Thanks eveyone!!

  • @blackpenredpen
    @blackpenredpen 3 года назад +172

    U have the coolest integral thumbnails on YT!

    • @nombreusering7979
      @nombreusering7979 3 года назад +2

      You did a video on this topic, I think
      The Bose integral

    • @MichaelPennMath
      @MichaelPennMath  3 года назад +52

      Thanks, I'll never tell my secret!! j/k: I hand place all of the characters using canva and use a color pallet generator to make sure the colors work together.

    • @nombreusering7979
      @nombreusering7979 3 года назад +2

      @@angelmendez-rivera351 Ofcouse
      I said he did a video about this "topic" as a whole

    • @wise_math
      @wise_math 3 года назад

      @@MichaelPennMath you only use canva for the thumbnails?

  • @Grassmpl
    @Grassmpl 3 года назад +25

    It is important to note that decomposing the infinite series into 2 parts trick only works due to absolute convergence.

  • @josebeleno1213
    @josebeleno1213 3 года назад +50

    In 12:05 the summation of the odd squares should go from 0 to infinity.

    • @MAULIKPATELnamste
      @MAULIKPATELnamste 3 года назад +13

      I too deal with this Trauma inflicted by Michael's mistake

    • @skulliam4
      @skulliam4 3 года назад +16

      Or use 2n-1

    • @xCorvus7x
      @xCorvus7x 3 года назад +4

      @skulliam4 That would be more elegant and, since he usually excludes zero from the naturals, a bit more consistent.

    • @josebeleno1213
      @josebeleno1213 3 года назад +1

      @@MAULIKPATELnamste jajajjsjsjjajajjs

    • @mirkorokyta9694
      @mirkorokyta9694 3 года назад +4

      It is important correct the mistake in the next step by making another small mistake "in the opposite direction" :)

  • @maxsch.6555
    @maxsch.6555 3 года назад +53

    That integral is related to a "special" version of the bose integral.
    We have for the bose integral:
    ∫₀∞ xⁿ/(eˣ-1) dx = Γ(n+1) ζ(n+1)
    We have an similar result if we switch the minus with a plus:
    ∫₀∞ xⁿ/(eˣ +1) = Γ(n+1) η(n+1)
    So for n = 1 we get the result in the video :)

    • @wansichen3743
      @wansichen3743 3 года назад

      ya that is what i was thinking too and i wonder if he will pull out polylog suddenly some how

    • @andyb2977
      @andyb2977 3 года назад +9

      To add, that "special" version is sometimes called the Fermi Integral or Fermi-Dirac integral, and it is related to the behavior of fermions in the same way that the Bose integral is related to bosons. And writing this comment is the first time I realized that bosons are named after Bose and fermions are named after Fermi.

  • @stevenpurtee5062
    @stevenpurtee5062 3 года назад +20

    At 12:05, I think you wanted (2n-1) in the first series. At n=1, the first term is 3 in the parentheses. You are leaving out 1.

    • @Ssweis23
      @Ssweis23 2 года назад +2

      You are correct, but in the next step he rewrites that sum as (sum of all n's) - (sum of all even n's) so it gets to the right answer

  • @michaelz6555
    @michaelz6555 3 года назад +5

    I was able to solve this using contour integration. I struggled with the integral form and the contour at first, but once I had them right the solution was straightforward. As a plus, it had advantage of not needing the Basel relation.

  • @nadeeraudayanga528
    @nadeeraudayanga528 3 года назад +8

    Rewrite the function to be integrated as x*exp(-x)/1+exp(-x), then apply infinite geometric series to the denominator and integrate. Much simpler I think!

    • @toddtrimble2555
      @toddtrimble2555 2 года назад

      That's also how I did it. I agree: simpler.

  • @domc3743
    @domc3743 3 года назад +14

    changing the denominator to e^x -1 yields a very interesting integral whose solution is that of the Basel Problem!
    i have unsuccessfully been trying to solve it using the Leibniz Rule. would be cool if you could do a video on it

    • @martinepstein9826
      @martinepstein9826 3 года назад +3

      I think the same method used in this video ought to work.

  • @surem8319
    @surem8319 3 года назад +4

    If found the general solution to this one by accident while working on another class of integrals :)
    The integral of x^n/(1+e^(bx) ) from 0 to infinity equals n!/b^(n+1) *((2^n-1)/2^n ) *ζ(n+1), where ζ(n) is the Riemann function. Pretty cool.
    Another cool exercise you could do is finding the value of the sum of the form:
    F(n)/n! from n=0 to n=infty
    Where F(n) represents the n'th Fibonacci number. The result is quite pretty (compared to what I expected at least).

  • @entropia5874
    @entropia5874 3 года назад +11

    What do mathematicians do when they meet on a conference?
    They integrate.

  • @DougCube
    @DougCube 3 года назад +4

    At 4:44, since the integrals are indefinite, don't you have to include a "+ C" then show that C=0 to set those equal like that? (It's an easy step by checking with u=0.)

  • @reijerboodt8715
    @reijerboodt8715 3 года назад +1

    Very nice video! Might have been nice to mention we can split the alternating sum into the positive and negative parts, because the sum absolutely converges (because it is the Basel sum). Keep it up Michael!

  • @G0r013
    @G0r013 3 года назад +1

    To split the summation in 12:24 it is important to notice that the original series converges absolutely

  • @umagan619
    @umagan619 3 года назад +4

    Another approach..expand the 1/(1+e^x) as a geometric series and use the gamma integral formula to simplify further..you ultimately evaluate the same sum anyways...

    • @faisalal-faisal1470
      @faisalal-faisal1470 3 года назад +2

      We can't expand 1/(1+e^x) as a geometric series because e^x>1 for x>0.
      However we have x/(1+e^x) = xe^(-x)/(1+e^(-x)) and now we can expand 1/(1+e^(-x)) as a geometric series to get x/(1+e^x) = sum_{n=0}^{\infty} (-1)^n x e^{-(n+1)x} dx, which we can integrate termwise because of uniform convergence.
      A quick integration by parts gives int_0^{\infty} x e^{-(n+1)x} dx = 1/(n+1)^2, so we get the same result at the end.

  • @menguistoluismatola-yk7nj
    @menguistoluismatola-yk7nj Год назад

    Thank you Michael Penn!

  • @davidgillies620
    @davidgillies620 3 года назад

    You can get the indefinite integral in a reasonably straightforward way using the polylogarithm function. The integral evaluates to x^2/2 - x ln(1 + exp(x)) - Li_2(-exp(x)). Using the power series definition of Li and the MacLaurin expansion of ln(1 +x) you can show that d/dx Li_2(-exp(x)) is -ln(1 + exp(x)) and then it's essentially integration by parts.

  • @krisbrandenberger544
    @krisbrandenberger544 3 года назад

    The summand for the sum of the squares of the reciprocals of the odd numbers should be 1/(2*n-1) in order to make the first term in the expansion of the summation 1.

  • @DeanCalhoun
    @DeanCalhoun 3 года назад +2

    love these type of integrals, great work

  • @Frank9412co
    @Frank9412co 3 года назад +1

    Good job as always, prof. Penn, I love the integrals which involves series. I sent you 2 months ago (IIRC) some problems coming from the colombian math olympiad and I'd love to see one of these problems solved on your channel. Keep up the good job!

    • @soutriksarangi5580
      @soutriksarangi5580 3 года назад

      Could u plz post them in the comment section, so that we can try :)

  • @buxeessingh2571
    @buxeessingh2571 3 года назад +2

    9:35-9:48 -- one of the problems with doing problems like this in Calculus 2 is that you have to hand wave past measure theoretic considerations. I used to say that non-mathematicians do it this way and hope that mathematicians can verify the witchcraft.

  • @renesperb
    @renesperb 2 года назад

    I suggest a somewhat different way:
    1/(Expx +1) = Exp[-x]/(1+Exp[-x])=Exp[-x]-Exp[-2x]+Exp[-3x] ...... Then calculate the Integral of x*Exp[-n x] from 0 to inf., which by Integration by parts gives 1/n^2 .The series you obtain has the value pi^2/12.

  • @lucagagliano5118
    @lucagagliano5118 3 года назад

    Integral and series cannot be exchanged in general. But in this case can be done because of uniform convergence of the geometric series.

  • @robertgerbicz
    @robertgerbicz 3 года назад +8

    At 1:36 use just the known:
    integrate (f'(x)/f(x))=log(abs(f(x))+C with f(x)=e^x+1.

    • @integralboi2900
      @integralboi2900 3 года назад +3

      He does the same thing, except he doesn’t plug it into that formula but derives it.

    • @pbj4184
      @pbj4184 3 года назад

      It's not magical. He shows where the property comes from

  • @hemdanemohamed1667
    @hemdanemohamed1667 3 года назад +5

    think there is a mistake in the odd sum, it should start from n=0.Anyway it doesn't make a change in the result.

    • @MrHK1636
      @MrHK1636 3 года назад

      Yeah you are right :) I was just about to comment on the same thing.

    • @curtmcd
      @curtmcd 3 года назад +2

      Or make it 2n-1 at 12:04

    • @DougCube
      @DougCube 3 года назад +1

      Actually, it would have changed the result, but he makes another mistake later that cancels out this mistake.

  • @factorization4845
    @factorization4845 3 года назад +1

    12:03 it has to be (2n-1)² because the 1 is excluded. But at the end it's still correct

    • @DougCube
      @DougCube 3 года назад

      I just spotted the same error. He makes another error later that cancels this one out.

  • @tomctutor
    @tomctutor 3 года назад

    Done a comparison test with ∫xexp(-x).dx, since 0≤x/(1+exp(x))≤x/exp(x).
    This integral ∫xexp(-x).dx between x=0 to x=∞ which can be easily shown to =1
    Therefore our problem integral converges to a value ≤1.
    I know doesn't solve it but at least gives you something to check you answer with!

  • @bilalabbad7954
    @bilalabbad7954 2 года назад

    I love your lessons

  • @abusoumaya8469
    @abusoumaya8469 3 года назад +4

    5:41
    girlfriend: we let U = x 😕🥲

  • @CTJ2619
    @CTJ2619 2 года назад

    More geometric puzzle problems please

  • @Hvidbergen
    @Hvidbergen 3 года назад +3

    At appeoximately 12:15, when splitting the sum into n even and odd, is there not a small mistake? The first term, when n=1, will not be included when summing from 1 with the expression (2n+1), which will start at 3. Shouldn’t the denominator be (2n-1) instead?

    • @tonyhaddad1394
      @tonyhaddad1394 3 года назад

      Exactly i realised too ( we re right ????)

    • @tonyhaddad1394
      @tonyhaddad1394 3 года назад

      n must be evaluated from 0 to i finity in the first part to get (1)

    • @MAULIKPATELnamste
      @MAULIKPATELnamste 3 года назад

      Well, Things clear up at 12:48 in the video, while splitting up the term.

    • @Hvidbergen
      @Hvidbergen 3 года назад

      @@MAULIKPATELnamste Doens't change the fact that you are still missing a +1 constant term, by neglecting the (2n+1) = 1 (when n=0) term, which evaluates to 1. So basically, the final answer should be 1 + pi^2 / 12, unless I am missing something.

    • @DougCube
      @DougCube 3 года назад

      @@Hvidbergen The correct answer is pi^2/12. What you're missing is that he went on to make another mistake that canceled out this mistake. It was on the very next step/line.

  • @jacklangner9867
    @jacklangner9867 3 года назад +1

    Excellent video! A suggestion for something different, find the nth derivative of 1/(1+x^2).

  • @thephysicistcuber175
    @thephysicistcuber175 3 года назад +1

    Michael Pls. For tool one just multiply and divide by e^(-x) and it becomes a trivial integral.

  • @mathsandsciencechannel
    @mathsandsciencechannel 3 года назад +1

    Thank you sir you are the inspiration behind maths and science channel

  • @rfsk45
    @rfsk45 3 года назад +3

    Just what I needed for a homework... thx

  • @anurupsil8216
    @anurupsil8216 3 года назад

    The first part integration 1 over e to the x +1 . I think it can be done in an another process by multiplying e to the neg x in both the numerator and denomination

  • @paulkohl9267
    @paulkohl9267 3 года назад

    Beautiful integral. Hash marks to denote end of proof reminds me of Ancient Egyptian where that represents the sound "ai". Excellent!

  • @zafarkamolov8162
    @zafarkamolov8162 3 года назад

    you earned my subscription!

  • @draaagoo7799
    @draaagoo7799 Год назад

    interesting!! i did a different approach. and that by multiplying first by exp(-x) and interpreting the quotient 1/1+exp(-x) as a geometric series and then proceeded by IBP...

  • @qaiserbozdar1542
    @qaiserbozdar1542 3 года назад

    Sir I am requesting you to solve a bunch of permutations, combinations and probability problems and also cover some Applied word problems in calculus.

  • @KaiqueSantos-xe1xu
    @KaiqueSantos-xe1xu 3 года назад

    What a beautiful integral

  • @irrelevant_noob
    @irrelevant_noob 3 года назад

    11:55 to 13:05 a literal example of two wrongs making a right: the sum for "2n+1" should've been from 0 to infinity. :-B

  • @CM63_France
    @CM63_France 3 года назад

    Hi,
    Great! I think I proofed the reverse of this : that the alternate sum of the n squared equals that integral.
    For fun:
    2 "so let's may be go ahead and",
    3 "ok, great".

  • @MoodyG
    @MoodyG 3 года назад

    I got an integral for you: integrate e^(-x-a/x)/(b+c/x) from 0 to +inf where a, b, and c are constants.

  • @stenzenneznets
    @stenzenneznets 3 года назад +1

    I would like to suggest this difficult problem:
    Show that the sum from n=1 to n=infinity of 1/(n^(1+|sin(n)|)) diverges.

    • @paul_w
      @paul_w 3 года назад +1

      I don't see how it diverges, |sin(n)| averages 0.6 and the zeta function converges for all n>1.

    • @stenzenneznets
      @stenzenneznets 3 года назад

      @@paul_w try it yourself with software

    • @paul_w
      @paul_w 3 года назад +1

      @@stenzenneznets I did and looking at the partial sums it seems convergent to around 3.3 . But obviously this is no proof, it could diverge very slowly. But again I don't see how it would diverge (cf previous post).

    • @stenzenneznets
      @stenzenneznets 3 года назад

      I can prove it diverges, but I world like to see how Michael manages to do it

  • @vardaandua3585
    @vardaandua3585 3 года назад

    An interesting one... wasn't able to solve it on my own... learnt something new.. thank you Michael

  • @holyshit922
    @holyshit922 3 года назад

    His way calculating integral Int(1/(e^x+1),x)
    reminds me how i calculated integral Int(x/sqrt(e^x+(x+2)^2),x)

  • @YK-xh3ls
    @YK-xh3ls 3 года назад +1

    why not just add and subtract xe^x on the numerator of the original integral, and then it becomes a simple integration by parts problem

  • @Tomaplen
    @Tomaplen 3 года назад

    Why not write constant of integration when finding the antiderivative? Or maybe why it has to be 0?

  • @tomatrix7525
    @tomatrix7525 3 года назад +1

    At 12:18, wouldn’t he also have to add 1, because the case when n=1 is not considered in his splitting of the sum? Edit: I’ve seen another comment regarding this, anothe solution would be to sum the odd terms from 0 to infinity instead. Either way, it’s a minor error but would result in an answer of pi^2/12 + 1. Could someone clear up if I am mistaken or he is?

    • @MAULIKPATELnamste
      @MAULIKPATELnamste 3 года назад

      At 12:48 onwards, the summations clear up automatically.

    • @tomatrix7525
      @tomatrix7525 3 года назад

      @@MAULIKPATELnamste yeah, thanks noticed that. I guess his error was more of a mis-use of the equals sign

    • @martinepstein9826
      @martinepstein9826 3 года назад

      @@tomatrix7525 He just made a small mistake writing the sum. No need to get complicated.

    • @DougCube
      @DougCube 3 года назад

      The correct answer is pi^2/12. What you're missing is that he went on to make another mistake that canceled out this mistake. It was on the very next step/line.

  • @nishatmunshi4672
    @nishatmunshi4672 3 года назад

    I think at 12:00 that sum will go from n=0 to infinity

  • @hamzatox
    @hamzatox 3 года назад

    Excellent. Suggestion: converge of sum(1/n^(2+cos(n))).

  • @keshavb3128
    @keshavb3128 3 года назад

    Hello Michael, I would like to suggest a problem from a 1962 Moscow Mathematical Olympiad: Given a_0, a_1, ... , a_n. It is known that
    a_0=a_n=0;a_k-1-2a_k+a_k+10 for all k = 1, 2, ... , k-1.
    Prove that all the numbers are nonnegative
    .
    Your videos on math are amazing!

    • @ricardocavalcanti3343
      @ricardocavalcanti3343 3 года назад

      It is not clear what "a_k-1-2a_k+a_k+10 for all k = 1, 2, ... , k-1" means.

    • @keshavb3128
      @keshavb3128 3 года назад

      @@ricardocavalcanti3343 It means a sub k, 2a sub k and so on.

    • @keshavb3128
      @keshavb3128 3 года назад

      In case what a_o or 2a_k means, it means a sub 0 and 2a sub k.

    • @ricardocavalcanti3343
      @ricardocavalcanti3343 3 года назад

      @@keshavb3128 But what is the condition on a_k-1-2a_k+a_k+10?

    • @keshavb3128
      @keshavb3128 3 года назад

      @@ricardocavalcanti3343 that a_0=a_n=0;a_k-1-2a_k+a_k+10 is only true when k=1,2,..., k-1 and are nonnegative.

  • @happyrogue7146
    @happyrogue7146 3 года назад

    Hey sorry if you only welcome integral suggestions but my suggestion is on number theory. prove that n^5 and n always have the same last digit for n being a positive integer. i've tried for the first 10 positive integers and it seems to hold, but i've failed to generalize.

  • @varundayawar5597
    @varundayawar5597 3 года назад

    Thank u mr helped me a lot🙏🏽🙏🏽🙏🏽🙏🏽🙏🏽🙏🏽🙏

  • @nevokrien95
    @nevokrien95 3 года назад

    I did it in 3 minites in my head using the gama and eta functions...

  • @doctorb9264
    @doctorb9264 3 года назад

    Proving Math is amazing. QED.

  • @tomatrix7525
    @tomatrix7525 3 года назад

    Hard integrals are always great

  • @bot24032
    @bot24032 3 года назад

    Integrate 1/x^5 and 1/(x^5+1)

  • @michaelz2270
    @michaelz2270 3 года назад

    Not to be one of "those people" (ok I am one of those people), but you are integrating xe^{-x} / (1 + e^{-x}), which expands to a geometric series sum 1 to infinity of (-1)^{n+1} x e^{-nx}. You can scale the integral of a given term by making the substitution u = nx, getting the sum from 1 to infinity of (-1)^{n+1}/(n^2) times the integral from 0 to infinity of ue^{-u}. The integral is 1 so you get just the sum from 1 to infinity of (-1)^{n+1}/(n^2) which is pi^2/12 as you do at the end there.

  • @johanndohmann1281
    @johanndohmann1281 11 месяцев назад

    ❤❤❤can you please make a video solving Int_0^inf x^3/(e^x-1) dx ??? it is used to derive Stefan-Boltzmann law from plancks law. I struggled with this. not able to crack the nut. The solution is (pi^4)/15. found by wolfram alpha, what is not realy satisfying. Guys like Planck and all these physicists of the early 1900 century are so talented, just like you Prof. Penn..... Best wishes from Germany

  • @lebuzzer9275
    @lebuzzer9275 3 года назад

    Hi ! Here's a quite quick problem :
    Find all integers n (if they exist) s.t. (2^{4n+2}+1)/65 is a prime number.
    Good luck !

    • @ricardocavalcanti3343
      @ricardocavalcanti3343 3 года назад

      Quick to state, not to solve! :-)
      Here is my solution:
      (1) For F(n) = (2^{4n+2}+1)/65 to be an integer (not necessarily prime), n must be of the form n=3k+1, where k is a nonnegative integer. Indeed, 2^{4(3k+1)+2}+1 = 2^{12k+6}+1 = 64^{2k+1}+1 = (-1)^{2k+1}+1 (mod 65) = 0 (mod 65).
      (2) Next, we note that 2^{12k+6}+1 = (2^{4k+2}+1)(2^{8k+4}-2^{4k+2}+1) = f(k)g(k). These factors have the following divisibility properties:
      (2.1) For any nonnegative k, f(k) is a multiple of 5: f(k) = 2^{4k+2}+1 = 4x16^{k}+1 = (-1)x1^{k}+1 (mod 5) = 0 (mod 5). Moreover, if k=3m+1, where m is a nonnegative integer, then f(k) is also a multiple of 65: f(3m+1) = 2^{4(3m+1)+2}+1 = 0 (mod 65) by (1).
      (2.2) If k=3m or k=3m+2, where m is a nonnegative integer, then g(k) is a multiple of 13. Indeed, g(k) = 2^{8k+4}-2^{4k+2}+1 = 16^{2k+1}-4x16^{k}+1 = 3^{2k+1}-4x3^{k}+1 (mod 13) = (3^{k+1}-1)(3^{k}-1) (mod 13). Therefore,
      (2.2.a) g(3m) = (3^{3m+1}-1)(3^{3m}-1) (mod 13) = (3x27^{m}-1)(27^{m}-1) (mod 13) = (3x1^{m}-1)(1^{m}-1) (mod 13) = 0 (mod 13);
      (2.2.b) g(3m+2) = (3^{3m+3}-1)(3^{3m+2}-1) (mod 13) = (27^{m+1}-1)(9x27^{m}-1) (mod 13) = (1^{m+1}-1)(9x1^{m}-1) (mod 13) = 0 (mod 13).
      Having established the preliminary results above, now we can show that there is no integer n such that F(n) = (2^{4n+2}+1)/65 is a prime number. For this, we have to examine the following cases:
      (3.1) If n0, it follows from (2.1) and (2.2.a) that F(n) = F(9m+1) = f(3m)g(3m)/65 = 5Mx13N/65 = MN, where M and N are integers greater than 1; therefore, F(n) is not prime in this case;
      (3.2.d) In a similar way, if k=3m+2 and m>=0, it follows from (2.1) and (2.2.b) that F(n) = F(9m+7) = f(3m+2)g(3m+2)/65 is not prime either;
      (3.2.e) Finally, if k=3m+1 and m>0, it follows from (2.1) that F(n) = F(9m+4) = f(3m+1)g(3m+1)/65 = 65MxN/65 = MN, where M and N are integers greater than 1; therefore, also in this case F(n) is not prime.
      These cases exaust all possibilities, thus proving that F(n) is not prime for any integer n.

    • @lebuzzer9275
      @lebuzzer9275 3 года назад +1

      @@ricardocavalcanti3343 Well done ! That was "the right way" to solve this exercise. (There were actually 2 intermediate questions that I deleted, otherwise it was too simple :-) ).

  • @tonyhaddad1394
    @tonyhaddad1394 3 года назад +1

    12:32 where is the number 1 in the odd set ??

    • @petersievert6830
      @petersievert6830 3 года назад +1

      has been pointed out by others, that it should start at n=0

  • @AaronRotenberg
    @AaronRotenberg 3 года назад

    3:02 Where do I buy the chalkboard with the self-rewriting checkbox?

  • @cicik57
    @cicik57 3 года назад

    why do you split series 1/(2n+1)^2 like that?

  • @admink8662
    @admink8662 3 года назад

    I thought you would differentiate the RHS of the first tool.

  • @pinum-enum
    @pinum-enum 3 года назад

    Mr Penn can you do a video about 1986 Q6 IMO please

  • @mikebrown4471
    @mikebrown4471 3 года назад

    Can you make a video about Glauert Integral?

  • @vitelot
    @vitelot 3 года назад

    What about using residues?

  • @tahasami3409
    @tahasami3409 3 года назад

    Good thank.......

  • @i.am.jihoonk
    @i.am.jihoonk 3 года назад +3

    nobody here?

  • @stevenpurtee5062
    @stevenpurtee5062 3 года назад

    Not to be that guy, but you forgot your limits on the integral at 7:36. I'm going to keep watching and see if you catch it. Yep, you caught it.

  • @Tis_Fari
    @Tis_Fari 3 года назад

    You've lost me when you started to solve the second question. That's a bit too advanced for me.

  • @backyard282
    @backyard282 3 года назад

    Is it possible to do this with contour integration as well?

    • @demenion3521
      @demenion3521 3 года назад

      in theory yes, but the problem is that the function has infinitely many poles (i*pi*(2k+1) for all integers k) and moreover the integral starts at 0, so you would have to find an appropriate contour that you can calculate.

  • @rudrarocks3056
    @rudrarocks3056 3 года назад

    I did it

  • @nicepajuju3900
    @nicepajuju3900 2 года назад

    Good.

  • @aashutoshgupta6690
    @aashutoshgupta6690 3 года назад

    These are jee mains questions 😂😁🙂

  • @abdellatif8554
    @abdellatif8554 3 года назад

    Hello, can you explique me the zeta function , thanks

  • @cernejr
    @cernejr 3 года назад

    Some 9 ads in 14min video :( . Got a headache from all those interruptions.

  • @Veldora900
    @Veldora900 3 года назад

    стендап не твое парень завязывай

  • @cuonghienthaosonbuitrung2841
    @cuonghienthaosonbuitrung2841 3 года назад +1

    im early too